Tai Lieu 2013-2014 PDF

You might also like

Download as pdf or txt
Download as pdf or txt
You are on page 1of 18

Bài giảng trường Đông 2013

DÃY SỐ VÀ CÁC TÍNH CHẤT SỐ HỌC


PHẦN LÝ THUYẾT

Trong phần này, ta xét dãy số  un  với u1 , u2   , un 2  aun1  bun , n  1 và c  u1u3  u22 .

Tính chất 1.

un 2 un  un21  ( b)n1 (u3u1  u22 ) với mọi số tự nhiên n.

Tính chất 2.

un21  c
Nếu b  1 thì ta có un 2  với mọi số tự nhiên n.
un

Tính chất 3.

Nếu u1 , u2 , a , b nguyên thì ( a 2  4b)un21  4( b)n c là số chính phương với mọi n.

Tính chất 4.

Xét dãy  vn  thỏa mãn: v1  u12 , v2  u22 , vn 2  ( a 2  2)vn1  vn  2c thì vn  un2 với mọi n
(trong trường hợp b  1) .

Chứng minh chi tiết cho các tính chất này có thể xem thêm trong bài viết:

“Khám phá một số tính chất của dãy truy hồi tuyến tính cấp 2”

trong tài liệu Chuyên đề Toán học số 9.

Bài tập rèn luyện.

Bài 1. Cho dãy số ( xn ) thỏa mãn x1  1, x2  1 và xn 2   xn1  2 xn , n  1, 2, 3,...

Xét dãy  vn  xác định bởi vn  2n1  7un21 , n  2, 3, 4,...

Chứng minh rằng mọi số hạng của dãy ( vn ) là số chính phương.

1
Bài 2. Cho dãy số  xn  xác định bởi x0  0, x1  1, xn 2  rxn1  xn , n  1 . Chứng minh

rằng x1  x3  x5  ...  x2 m1  xm2 với mọi số nguyên dương m.

Bài 3. Cho số nguyên dương m. Xác định dãy số ( xn ) như sau:

x0  0, x1  m, xn1  m2 xn  xn1 , n  1, 2, 3,...

Chứng minh rằng với mọi cặp số tự nhiên ( a , b) với a  b là nghiệm của phương trình
a2  b2
 m khi và chỉ khi ( a , b)  ( xn , xn1 ) với n là số tự nhiên nào đó.
ab  1

Bài 4. Cho dãy số ( xn ) xác định bởi x0  0, x1  1, xn 2  2 xn1  xn  1, n  1, 2, 3,...

Chứng minh rằng 4 xn xn 2  1 là số chính phương với mọi n.

u  1, u2  2,
Bài 5. Cho dãy số (un ) với  1 .
un  2
 4un  1
 un
, n  1

Tìm tất cả các giá trị của n sao cho un  1 là số chính phương.

u  1, u1  9,
Bài 6. Cho dãy số (un ) xác định bởi  0
un 2  10un1  un , n  1, 2, 3,...

5un1  un 3un2  1
Chứng minh rằng và là các số nguyên với mọi n.
4 2

Bài 7. Cho dãy số nguyên  an  xác định bởi công thức truy hồi sau

an21  4
a1  1, a2  5, an  với n  3 .
an 2

Chứng minh rằng tất cả số hạng của dãy đều là số nguyên.

Bài 8. Tìm tất cả các dãy vô hạn các số nguyên dương  an  bị chặn và thỏa mãn

an1  an 2
an  , n  3.
 an1 , an2 

2
Lời giải.

Đặt yn   an , an1  , n  1, 2, 3,... Ta thấy rằng yn1 chia hết cả un1 và un 2 nên cũng chia
hết yn an 2  an1  an .

Suy ra yn1 chia hết cả an1 , an nên yn1 | yn . Do đó,  yn  là dãy không tăng của các số
nguyên dương và kết thúc là dãy hằng với hằng số y nào đó.

Xét dãy khi yn  y : yan  an1  an 2 . Ta xét các trường hợp:

- Nếu y  1, an  an1  an 2 thì lim an   nên dãy không bị chặn và mâu thuẫn.

an1  an 2 an1  an 2


- Nếu y  3 thì an    max a n1 , an 2  .
y y

Tương tự, an1  max an , an1   max an 2 , an1  nên max an , an1   max an1 , an 2  và dãy
này vô hạn, mâu thuẫn với tính chất nguyên dương của dãy.

 1 
- Nếu y  2 thì 2an  an1  an 2 nên an  an1     an1  an 2  nên suy ra hiệu an  an1
 2 
tiến về 0 nên ai là dãy hằng với i đủ lớn.

Hơn nữa, từ 2 an  an1  an 2 , ta có 2   an , an1  nên ai  2 với mọi i  i0 đủ lớn.

Nếu an  an1  2 với n  1 thì  an1 , an    an1 , 2   1 hoặc 2 mà

an1  an
2  an1   a  2.
 an1 , 2  n1
Từ đó suy ra an  2 với mọi n và dãy số này thỏa mãn đề bài.

Bài 9. Cho  an  là các số nguyên dương lẻ xác định như sau

a1  r , a2  s và an là ước dương lẻ lớn nhất của an1  an 2 .

Chứng minh rằng với n đủ lớn thì an là hằng số và xác định hằng số đó theo r , s.

3
Lời giải.

Kí hiệu s(n) là ước nguyên dương lẻ lớn nhất của n. Với m là số nguyên dương lẻ, ta
có s(2t m)  m với t nguyên dương.

mn
Suy ra với m , n lẻ thì m  n 2 và s( m  n)  .
2

Đặt Mn  max an , an1  với n  1, 2, 3,...

an  an1 1  an  an1 
Ta có an 2  s( an  an1 )  và an 3  s( an 2  an1 )    an1  .
2 2 2 

Do đó Mn 2  Mn và đẳng thức xảy ra khi và chỉ khi an  an1 . Do Mn  1 nên với n đủ
lớn thì Mn là dãy hằng, tức là n0 sao cho an0  an0 1  c , quy nạp thì suy ra được rằng
an  c , n  n0 .

Mặt khác, đặt d   r , s  thì  an , an1   d với mọi n nên c  d.

Vậy tồn tại n đủ lớn sao cho an là dãy hằng và khi đó an   r , s  .

Bài 10. Cho dãy số nguyên dương ( an ) thỏa an  an1  1. Đặt bn  a1  a2  ...  an . Chứng
minh rằng luôn có một số chính phương trong dãy sau bn , bn  1, bn  2,..., bn1  1.

Lời giải.

Giả sử rằng không có số chính phương nào trong dãy bn , bn  1, bn  2,..., bn1  1 , tức là
2
tồn tại m nguyên dương sao cho m 2  1  bn , bn1  1   m  1  1 hay

bn1  bn  1.

Ta sẽ chứng minh bằng quy nạp rằng bn1  bn  1 với mọi số nguyên dương n.

2
Suy ra bn1  bn  2 bn  1  an1  2 bn  1  4  a1  a2  ...  an1    an  1 .

Bằng quy nạp, ta dễ dàng có được đpcm.

4
DÃY SỐ VÀ TÍNH TUẦN HOÀN.

 x  a , x2  b , x3  c ,
Bài 1. Với a , b là các số nguyên dương, xét dãy số  1 .
 xn1  5xn  7 xn1  xn 2 , n  3

Chứng minh rằng với mọi cách chọn các số a , b , c thì dãy số đã cho hoặc tồn tại vô số số
hạng chia hết cho 2013 hoặc không tồn tại số hạng nào chia hết cho 2013.

Bài 2. Cho p là số nguyên tố và a , b là hai số nguyên thỏa mãn a 2  ab  b2 không chia


 v  a , v2  b ,
hết cho p. Xét dãy số  1 .
vn 2  vn  vn1 , n  1

Chứng minh rằng dãy vn mod p tuần hoàn và chu kì không phụ thuộc vào p.

Bài 3. Cho một số nguyên tố lẻ p thỏa mãn 2 h  1(mod p) với mọi h  p  1, h   * và


p
một số chẵn a  ( ; p) . Xét dãy số ( an ) xác định bởi: a0  a , an1  p  bn , n  0,1, 2,... với bn
2
là ước số lẻ lớn nhất của an .

Chứng minh rằng ( an ) là dãy số tuần hoàn và tìm chu kì dương nhỏ nhất của nó.

Lời giải.

Ta thấy rằng với mọi i, ai là số chẵn, ai  p  bi 1  p mà

ai 1 p p
bi 1   , i  ai  p  bi 1  .
2 2 2
p
Suy ra  ai  p, i , tức là giá trị của các số hạng của dãy (an ) đã cho là hữu hạn.
2

Đặt ai  2ki  bi với ki , bi  , ki  0 và bi là số lẻ. Nếu bi  b j thì ai | a j hoặc a j | ai , theo


nhận xét trên thì ai  a j vì nếu ngược lại thì cả hai số này không thể nằm trong khoảng
p  p 
 , p  được. Cũng theo nhận xét trên thì phải tồn tại các giá trị i  j; i, j   , p  thỏa
2  2 
mãn ai  a j ; mặt khác ai  p  bi 1 , a j  p  b j 1 nên bi  b j , do đó ai 1  a j 1 , tiếp tục quá
trình này, ta được a0  a j i , tức là dãy đã cho tuần hoàn từ số hạng đầu tiên. Nếu đặt
T  j  i thì T chính là chu kì dương nhỏ nhất cần tìm của dãy đã cho.

5

n
Ta biết rằng lũy thừa của 2 trong n ! là   2  nên nếu đặt K là tích của tất cả các số tự
i 1
i

p  ( p  1)!
nhiên trong khoảng  , p  , tức là K  , thì lũy thừa của 2 trong K là
2   p 1 
 !
 2 

  p  1   p  1  p  1
  
i 1

2i   2i 1  

2
, đó cũng chính là lũy thừa của 2 trong tích của tất cả các số

p  p 1
chẵn trong khoảng  , p  . Suy ra k1  k2  k3  ...  kT  . Đẳng thức xảy ra khi và
2  2
p 
chỉ khi tập hợp a1 , a2 , a3 ,..., aT chính là tập hợp tất cả các số chẵn trong khoảng  , p  .
2 

Ta có a1a2 a3 ...aT  2k1  k2  k3 ... kT  b1b2b3 ...bT (mod p ) .

Theo giả thiết thì ai  bi 1 (mod p) nên a1a2 a3 ...aT  (1)T  b0b1b2 ...bT 1 (mod p ) , do cách chọn
T nên b0  bT , tức là a1a2 a3 ...aT  (1)T  b1b2 ...bT (mod p ) , suy ra:

(1)T  b1b2 ...bT  2k1  k2  k3 ... kT  b1b2b3 ...bT (mod p )  (1)T  2k1  k2  k3 ... kT (mod p )

do (bi , p)  1  (b1b2b3 ...bT , p )  1 . Ta được 22( k1  k2  k3 ... kT )  1(mod p) .

Hơn nữa, cũng theo giả thiết thì 2h  1(mod p), h  1, 2,3,..., p  2 nên từ hệ thức trên, suy
p 1
ra 2(k1  k2  k3  ...  kT )  p  1  k1  k2  k3  ...  kT  .
2
So sánh hai bất đẳng thức trên về tổng k1  k2  k3  ...  kT , ta được

p 1
k1  k2  k3  ...  kT  .
2

p 
Do đó a1 , a2 , a3 ,..., aT chính là tất cả các số chẵn trong khoảng  , p  , suy ra
2 

1 p  1 p  1
T  ( p  1)   .
2 2  4

 p  1
Vậy dãy số đã cho tuần hoàn với chu kì dương nhỏ nhất là T   .
 4 

 x  a , x1  b   ,
*
Bài 4. Cho dãy số  xn  thỏa mãn  0
 xn 2  xn xn1  1, n  1.
6
Chứng minh rằng nếu p là số nguyên tố và x2006 chia hết cho p thì

a. Tồn tại j  2006 mà x j chia hết cho p.

b. Tồn tại T  N * sao cho với mọi n  2006 thì xT  n  xn (mod p) .

c. Tồn tại j  2006 mà x jj chia hết cho x2006


2006
.

Lời giải.

a. Chú ý rằng nếu (n, p)  1 thì tồn tại m sao cho mn  1(mod p) .

b. Sử dụng kết quả của câu a.

c. Giả sử x2006  p1a1 p2a2 ...pkak , với mọi i  1, 2, 3,..., k thì theo câu b, tồn tại Ti sao cho
x2006  lTi  0(mod p) . Đặt T  [T1 , T2 , T3 ,..., Tk ] thì x2006  lT  0(mod p1 p2 ...pk ) .

Chọn l đủ lớn sao cho 2006  lT  max 2006a1 , 2006a2 ,..., 2006ak  thì dễ dàng có
2006  lT
được x2006  lT
 x2006
2006
. Đây chính là đpcm.

SỬ DỤNG CÔNG THỨC TỔNG QUÁT

Bài 1.

u  7, u2  50,
Cho dãy số  1 .
un1  4un  5un1  1975

Chứng minh rằng u1996 chia hết cho 1997.

Bài 2.

u0  2,
Cho dãy số (un ) thỏa mãn  2
un1  4un  15un  60 , n  1

1
Chứng minh rằng số  u  8  có thể biểu diễn thành tổng bình phương của ba số tự
5 2n
nhiên liên tiếp.
7
Bài 3.

u  0, u2  1,
Cho dãy số  un  thỏa mãn  1 .
un 2  un  un1  1, n  1

Chứng minh rằng với p là số nguyên tố lớn hơn 5 thì up (up 1  1) chia hết cho p.

Bài 4.

a  1, a1  1,
Cho dãy số nguyên ( an ) thỏa mãn  0 .
a
 n  6 an 1
 5 an 2
, n  2

Chứng minh rằng a2012  2010 chia hết cho 2011.

Bài 5.

a  1, a2  2011,
Cho dãy số ( an ) thỏa mãn  1 .
an 2  4022 an1  an , n  1.

a2012  1
Chứng minh rằng là số chính phương.
2012

Lời giải. Ta xét bài toán tổng quát sau

Cho p là số nguyên dương lẻ lớn hơn 1.

 x1  1, x2  p,
Xét dãy số nguyên dương  xn  được xác định bởi 
 xn  2  2 pxn 1  xn , n  
*

x p 1  1
Chứng minh rằng là số chính phương.
p 1

Bài toán này có một số lời giải như sau.

Cách 1 (dùng công thức tổng quát của dãy và biến đổi trực tiếp).

Phương trình đặc trưng của dãy số đã cho là t 2  2 pt  1  t 2  2 pt  1  0 có


  p 2  1  0 nên phương trình có hai nghiệm là t1  p  p 2  1, t2  p  p 2  1 .

Công thức tổng quát của dãy đã cho là xn  At1n  Bt2n , n  1, 2,3,...
8
Thay n  1, 2 tương ứng với hai số hạng cho trước của dãy, ta được hệ phương trình
sau:

 At1  Bt2  1
 2 2
 At1  Bt2  p

t2 t1 t1n 1  t2n 1
Giải hệ này, ta thu được A  , B  hay xn  , n  1, 2,3,...
2 2 2
2
t1p  t2p  2  t1  t2 
p /2 p /2
x p 1  1
Suy ra   .
p 1 2( p  1) 2( p  1)

Chú ý rằng t1  t2  2 p, t1t2  1 nên t1  t2  t1  t2  2 t1t2  2( p  1) .

Hơn nữa, ta cũng có Sn  t1n  t2n  , n  1 vì S1 , S2   và Sn  2  2 pSn 1  Sn .

Đặt t1  a, t2  b thì a  b  2( p  1), ab  1 và t1p /2  t2p /2  a p  b p .

p 1 p 1
p p i i p 1 i
Ta có a  b  (a  b) (1) a b  2( p  1)  (1)i a i b p 1i . Xét biến đổi sau:
i 0 i 0

p 3
p 1 2 p 1 p 1 p 1

 (1)i aib p 1i   (1)i aib p 1i  (1)


i 0 i 0
2
(ab) 2
  (1) a b
p 1
i i p 1 i

i
2
p 3 p 3
2 p 1 p 1 2 p 1 2 i p 1 p 1 2 i p 1
i i p 1 2 i i i p 1 2 i i i
  (1) (ab) b   (1) (ab) a  (1) 2
  (1) t 2
2
  (1) t 1
2
 (1) 2

i 0 p 1 i 0 p 1
i i
2 2
p 3 p 3
p 1 2 i p 1 2 i p 1 p 1
i  
2 2

  (1)   (1) S p 1 2i  (1)  N  


i
  (1) t1 2
 t2 2 2 2

i 0   i 0 2

2
Do đó t1p /2  t2p /2  N 2( p  1)   t1p /2  t2p /2   N 2  2( p  1)  .

N 2  2( p  1) 
x p 1  1
Vậy   N 2 là số chính phương. Ta có đpcm.
p 1 2( p  1)

Cách 2. (Xét dãy số phụ và dùng quy nạp).

9
Ta thấy rằng

x2  1 p  1
x2  p    1,
p 1 p 1
2 3 x4  1 4 p3  3 p  1
x3  2 p  1, x4  4 p  3 p    (2 p  1) 2 ,
p 1 p 1
x 1
x5  8 p 4  8 p 2  1, x6  16 p5  20 p3  5 p  6  (4 p 2  2 p  1) 2 .
p 1

Từ công thức truy hồi là xn  2  2 pxn 1  xn , ta có 2 pxn1  xn  xn 2 . Suy ra

xn  2  2 p  2 pxn  xn1   xn  4 p 2 xn  2 pxn 1  xn  (4 p 2  2) xn  xn 2 , n  3 .

x2 n  1
Ta sẽ xây dựng công thức của dãy yn  , n  1, 2,3,... và chứng minh các số hạng
p 1
của dãy này đều nguyên.

 y1  1, y2  2 p  1
Xét dãy  yn  thỏa mãn  với a, b được chọn sau.
 yn 2  ayn 1  byn , n  1

Do y3  4 p   2 p  1 nên a(2 p  1)  b  4 p 2  2 p  1 ; ta có thể chọn a  2 p, b  1 .

 y1  1, y2  2 p  1
Dãy số tương ứng là  .
 yn 2  2 pyn 1  yn , n  1

x2 n  1
Ta sẽ chứng minh bằng quy nạp rằng yn2  , n  1 . (*)
p 1

Với n  1, 2 , khẳng định (*) đúng.

x2 n  1 2 x 1
Giả sử ta có yn2  , yn 1  2 n  2 .
p 1 p 1

Ta có yn 2 yn  yn21  (2 pyn 1  yn ) yn  yn1 (2 pyn  yn1 )  yn 1 yn 1  yn2 , n .

Hơn nữa y3 y1  y22  2 p  2 nên yn 2 yn  yn21  2 p  2, n .

Từ công thức xác định dãy thì

10
yn  yn  2  2 pyn 1  yn2  yn2 2  2 yn yn  2  4 p 2 yn21  yn2  yn2 2  2  2 p  2  yn21   4 p 2 yn21 hay

x2 n  2  1 x2 n  1
yn2 2  (4 p 2  2) yn21  yn2  4( p  1)  (4 p 2  2)   4( p  1)
p 1 p 1
(4 p 2  2) x2 n  2  x2 n  1 x2 n  4  1
 
p 1 p 1

Khẳng định (*) cũng đúng với n  2 . Theo nguyên lí quy nạp, (*) được chứng minh.

xn  1
Do đó, ta đã chứng minh được với mọi n chẵn thì là số chính phương; nói riêng,
p 1
x p 1  1
ta cũng có cũng là số chính phương. Ta có đpcm.
p 1

DỰ ĐOÁN VÀ QUY NẠP.

Bài 1.

Cho dãy số nguyên dương a1 , a2 , a3 ,... thỏa mãn a1  1 và

a  2 if a n  2  a1 , a2 ,..., an  , an  2  0


an1   n .
an  3, otherwise

Chứng minh rằng với mọi số nguyên dương k thì tồn tại sốn nguyên dương n sao cho

an  k 2  an1  3.

Lời giải.

Ta tính được

n 1 2 3 4 5 6 7 8 9 10 11

an 1 4 2 5 3 6 9 7 10 8 11

Ta thấy rằng a1 , a2 ,..., a10  là một hoán vị của 1, 2, 3,...,10 và a11  11 nên bằng quy
nạp, ta chứng minh được rằng a10 k 1  10 k  1, a10 k  2  10 k  4 .

11
Suy ra với mọi số nguyên dương k đều xuất hiện đúng một lần trong dãy đã cho.

Hơn nữa, các số chính phương đồng dư với 0,1, 4, 5,6,9 theo modulo 10 nên ta có đpcm.

Bài 2.

(2  3)n  (2  3)n
Cho dãy số un  với n  0.
2 3
a. Chứng minh rằng tất cả số hạng của dãy đều nguyên.
b. Tìm tất cả các số hạng chia hết cho 3 của dãy đã cho.

Bài 3.

a0  0, a1  1,
Cho dãy số  an  xác định bởi  n
an1  3an  an1  2  1 , n  1

Chứng minh rằng an là số chính phương với mọi n.

Bài 3.

a  a2  1,
Cho dãy số ( an ) thỏa mãn  1 .
an1  7 an  an1 , n  2

Chứng minh rằng với mọi số nguyên dương n thì an  an1  2 là số chính phương.

Bài 4.

a  0, a1  1, a2  2, a3  6,
Cho dãy số  1
an 4  2an 3  an 2  2an1  an , n  1.

Chứng minh rằng với mọi n  1 thì an chia hết cho n.

Gợi ý. Chứng minh bằng quy nạp rằng an  nFn với Fn là số Fibonacci thứ n.

MỘT SỐ BÀI TOÁN KHÁC

Bài 1.

12
a0  a1  1,

Cho dãy số  (2n  3)an1  3nan1 .
an1  ,n  1
 n 3

n
Chứng minh rằng an 2  an1   ak an k đvới mọi số tự nhiên n thì an là số nguyên.
k 0

Lời giải.

Xem thêm lời giải chi tiết (bằng tiếng Hungary) tại đây:

http://www.komal.hu/verseny/2000-03/A.h.shtml

Bài 2.

a1  2, a2  7,

Cho dãy số nguyên ( an ) thỏa mãn  1 an2 1

 2  a n 1
  , n  2, 3, 4,...
 an1 2

Chứng minh rằng với mọi n  1 thì an là số lẻ.

Bài 3.

Cho một dãy vô hạn các số nguyên dương ( an ) được xác định như sau

an1  an  bn  n  1, 2,... , bn là chữ số tận cùng của an .

Tìm điều kiện cần và đủ của a1 để trong dãy số đã cho chứa vô hạn lũy thừa của của 2.

Lời giải.

Phần thuận:

Do a1 không chia hết cho 5 nên an không chia hết cho 5 với mọi n . Có nhận xét sau:

Chữ số tận cùng của an đến một lúc nào đó sẽ tuần hoàn với chu kì 2,4,8,6 . Do đó

 4| am 2
 
am  4 s  am  20s m , s   . Vì 2| am thì 
4| a
nên tồn tại vô hạn m để am  4l ( l
 m 4

13
không chia hết cho 5). Suy ra am 4 s  4  n  5s  . Mà l không chia hết cho 5 nên tồn tại vô

hạn k sao cho: 2 k  l  mod 5  hay tồn tại vô hạn k , s : 2 k  l  5s .

Phần đảo:

Tồn tại vô số số hạng của dãy là lũy thừa của 2. Giả sử 5|a1 thì 5| an , n (mâu thuẫn vì

2 k không chia hết cho 5).

Vậy điều kiện cần và đủ cần tìm là a1 không chia hết cho 5.

Bài 4.

Cho n  2 là số tự nhiên. Chứng minh rằng tồn tại hữu hạn các bộ n số nguyên dương
a , a
1 2
,..., an  thỏa mãn đồng thời các điều kiện sau:

(i) a1  a2  a3  ...  an .

(ii)  a1 , a2 , a3 ,..., an   1 .

n
(iii) a1    ai , ai 1  với quy ước an1  a1 .
i 1

Lời giải.

Đặt di  ai , ai1  , i  1,2, 3,..., n trong đó an1  a1 . Theo giả thiết, ta thấy rằng,

a1  d1  a2 , a2  d2  a3 ,..., an1  di1  an , i  1, 2, 3 .

Đưa vào dãy đã cho, ta được

d1  d2  ...  dn  a1  d1  a2  d1  d2  a2  ...  d1  d2  ...  dn1  an

Từ đây suy ra dn  an , nhưng do dn  an , a1  nên dn  an hay đẳng thức phải xảy ra.

Do đó, ta có ai  ai1  di , i  1, n . Tiếp theo, ta sẽ chứng minh rằng tồn tại vô số dãy số
thỏa mãn đề bài. Ta thấy rằng với dãy số thỏa mãn đề bài, tồn tại hữu hạn các số
14
ri
nguyên dương ri sao cho ai  a , i  1, n . Do a1 , a2 ,..., an   1 nên
ri  1 i1

a2 , a3 ,..., an   1 và an | a1 . Ta sẽ chứng minh bài toán bằng quy nạp

- Với n  2, do a2 | a1 nên ta có thể viết a2  m , a1  km với k , m   . Hơn nữa, do


a1 , a2   1 nên m  1 mà a1   a1 , a2   a2 , a1   2 nên với n  2 , tồn tại đúng một dãy
thỏa mãn đề bài.

- Giả sử bài toán đúng với n 1 , ta sẽ chứng minh rằng tồn tại hữu hạn dãy số thỏa
mãn trong trường hợp n phần tử.

Ta có hai trường hợp:

p
+ Nếu a2 không chia hết cho an , đặt a2  an với p , q   và  p , q  1, q  1 .
q

r1 p r1
Do a1  a2 , an | a1 nên suy ra là số nguyên dương. Với mỗi a2 , tồn tại hữu
r1  1 q r1  1
hạn các giá trị r1 và cũng do có hữu hạn các dãy n 1 thỏa mãn nên từ đó suy ra trong
trường hợp này, số bộ n phần tử là hữu hạn.

+ Nếu a2 chia hết cho an thì theo nguyên lí quy nạp, tồn tại hữu hạn dãy a2 , a3 ,..., an và
n
do cách xác định a1    ai , ai1  , ta thấy rằng tồn tại hữu hạn dãy số thỏa mãn đề bài.
i 1

Do đó, theo nguyên lí quy nạp, bài toán được chứng minh.

Bài 5.

Tìm tất cả các dãy vô hạn các số nguyên dương a1 , a2 , a3 ,... thỏa mãn đồng thời các tính
chất sau:

i) Dãy số đã cho tăng thực sự.

ii) Không có các số nguyên dương i , j , k nào, không nhất thiết phân biệt, thỏa mãn
ai  a j  ak .

15
iii) Tồn tại vô hạn các số nguyên dương k sao cho ak  2 k  1.

Gợi ý.

Đặt a1  c . Ta chứng minh được rằng ak  c  ak  2c bằng phản chứng.

Ta có amc 1  (2m  1)c và với mọi n  mc  1 , đặt n  qc  r ,0  r  c  1 thì

aqc  r  a( q 1)c  r  ...  amc 1  r  1  2(q  m)c  2(qc  r )  1 .

Do đó, nếu ak  2 k  1 thì bất đẳng thức trên đúng.

Nếu c  1 thì a2  c  1, ac  2c  1, ac 1  3c , mâu thuẫn nên c  1 và am  a( m1) c  r  2 m  1.

Bài 6.

Cho a1 , a2 với a1  a2 là các số nguyên cho trước. Với n  3, số an là số nguyên dương


nhỏ nhất lớn hơn an 1 và an có thể được biểu diễn một cách duy nhất dưới dạng tổng
của ai  a j mà 1  i  j  n  1 .

Chứng minh rằng nếu tồn tại hữu hạn các số chẵn trong dãy an  thì dãy an1  an 
tuần hoàn, nghĩa là tồn tại số nguyên dương T mà an 1T  an T  an 1  an , n .

Lời giải.

Cách 1.

Xét an là số lẻ với n  N . Với n đủ lớn, ta thấy rằng an  ak  al với k  l với k  N ,


điều này có nghĩa là :

(1) k  N  al  an  N  l  n  N .

(2) Nếu k  l khi cả hai số này đủ lớn thì al i  ak i  d , i  1, 2,3,..., N khi


al  N 1  ak  N 1  d .

(3) Với n đủ lớn thì ta có an1  an  aN . Xét

f :  N   N :  x1 , x2 , x3 ,..., xN   0, x2  x1 , x3  x1 , xN  x1  .

16
Nếu f an , an1 ,..., an N 1   b1 , b2 ,..., bN  thì bi   N 1 aN , i  1, 2,3,..., N .

Theo nguyên lí Dirichlet thì tồn tại số nguyên dương n, T sao cho

f  anT , anT 1 ,..., anT N1   f an , an1 ,..., anN1  , ở đây ta gia sử n đủ lớn.

Điều này có nghĩa là anT i  ani  d , i  0,1, 2,..., N  1 .

Theo (2) thì anT N  anN  d .

Từ đó suy ra amT  am  d , m  n  m  n, am1T  amT  am1  am .

Ta có đpcm.

Cách 2.

Do dãy số đã cho chỉ chứa hữu hạn số chẵn nên ta có thể giả sử chúng là

ai1  ai2  ai3  ...  aim .

Khi đó, tồn tại một số N sao cho với mọi r  N thì ar là số lẻ, tuy nhiên ar  ai  a j nên
hai số ai , a j khác tính chẵn lẻ.

Điều này có nghĩa là ar 1  ar  aim với mọi r  N .

Với n  N xét tập hợp

Sn  an  an1 , an  an2 ,..., an  ank  với k là số nguyên dương lớn nhất thỏa mãn
an  ank  aim .

Khi đó, rõ ràng ta có k  aim và từ an  an j bị chặn nên tồn tại hữu hạn tập hợp Sn .

Các tập hợp Sn có thể xác định được các giá trị is thỏa mãn an1  ais  a j nên Sn được
xác định duy nhất xác giá trị an1  an và xác định duy nhất tập hợp Sn1 .

Do có hữu hạn tập hợp Sn nên dãy Sn  nêu trên là dãy lặp hay dãy an1  an  cũng là
dãy lặp với các giá trị n  N đủ lớn. Ta có đpcm.

17
Bài 7.

Xét dãy số  an  ,  bn  xác định như sau:

- an là số các chỉ số k mà Cnk  1 (mod 3) .

- bn là số các chỉ số k mà Cnk  2 (mod 3).

Chứng minh rằng an  bn  2cn với mọi n. Tìm công thức tổng quát của  cn  .

Lời giải.

s s
Theo định lí Lucas nếu n   ai 3i , k   bi 3i thì Cnk  C ab00 C ab11 ...Cabss (mod 3).
i 1 i 1

Khi đó Cnk  0(mod 3)  ai  bi (i  0,1, 2,..., s) . Do đó,

ai  0  bi  0

C  1(mod 3)  ai  1  bi  0,1
k
n
a  2, co mot so chan cac so b_i
 i

ai  0  bi  0

C  2(mod 3)  ai  1  bi  0,1
k
n
a  2, co mot so le cac so b_i
 i

Nếu n có m vị trí mà ai  2 , t vị trí ai  0 và s vị trí ai  1 thì

m m
s ms t
an   C 2
m
2 , bn   Cms 2 m s 2t .
s  0 ,s  0(mod 2) s  0 ,s 1(mod 2)

 m m 
Từ đó suy ra an  bn  2t   C s ms
m
2   C ms 2 m s   2t .
 s0 ,s 0(mod 2) s  0 ,s 1(mod 2) 

Đồng thời cn chính là số vị trí bằng 1 trong biểu diễn tam phân của n.

18

You might also like